fereidoon

Active Member
ارسال ها
447
لایک ها
132
امتیاز
43
پاسخ : ماراتن جبر (سطح پیشرفته)

خيلي ممنون،اصلاح شد.
 

mojtabaaa1373

Active Member
ارسال ها
362
لایک ها
74
امتیاز
28
پاسخ : ماراتن جبر (سطح پیشرفته)

تا به حال دیده بودید smv و حسابی هندسی با هم ترکیب بشن و فقط با استفاده از مفهوم این دو تا یه سوال حل بشه؟احتمالا اساتید دیدن اگه یه وقت کسی ندیده بود حال ببینید!!!!!!!!!!!!:58:


برای این که فقط یه سوال تو ماراتن هم باشه سوال استاد فریدون:به جای e سیگمای 1 تقسیم بر kفاکتوریل رو بزارید و استقرا بزنید ببخشید یکم مختصر نوشته بودم.:eek:
 
آخرین ویرایش توسط مدیر

mahanmath

New Member
ارسال ها
898
لایک ها
701
امتیاز
0
پاسخ : ماراتن جبر (سطح پیشرفته)

این سوال رو باید تو ماراتن نابرابری می ذاشتی ....
SOS خیلی راحت این مسله رو حل می کنه چون همه ی S_a , S_b , S_c مثبتن !:98:

یکی سوال جدید بزاره لطفا ....
 

mahanmath

New Member
ارسال ها
898
لایک ها
701
امتیاز
0
پاسخ : ماراتن جبر (سطح پیشرفته)

این 2 تا سوال رو استاد Shoki فرمودند من پست کنم . منم بدون هیچ تغییری گذاشتمشون :)

1- tamame adade
ra biyabid k
baraye binahayatta m tavane m+1-om kamel bashad.
2-tamame adade
ra biyabid k
baraye binahayatta m tavane m-om kamel bashad.
 

hkh74

New Member
ارسال ها
213
لایک ها
392
امتیاز
0

ali_irysc

New Member
ارسال ها
319
لایک ها
294
امتیاز
0
پاسخ : ماراتن جبر (سطح پیشرفته)

سلام
می خواستم بگم چند تا از نامساوی مثل این بزارید بد نیست

متغیر ها مثبتن
 
آخرین ویرایش توسط مدیر

math

New Member
ارسال ها
1,129
لایک ها
1,096
امتیاز
0
پاسخ : ماراتن جبر (سطح پیشرفته)

ببخشید یک سوال داشتم

برای اعداد صحیح تعریف شده

1)
صحیح است

2)


3)


4)

ثابت کنید



من این سوال رو با تابع ضربی حل کردم 99.99 درصد جوب زدم چون اگر تابع ضربی رو بنویسیم دیگه نیازی به شرط 4 نیست:105:

tst 2012 Puerto Rico
 

ash1374

New Member
ارسال ها
253
لایک ها
422
امتیاز
0
پاسخ : ماراتن جبر (سطح پیشرفته)

ببخشید یک سوال داشتم

برای اعداد صحیح تعریف شده

1)
صحیح است

2)


3)


4)

ثابت کنید



من این سوال رو با تابع ضربی حل کردم 99.99 درصد جوب زدم چون اگر تابع ضربی رو بنویسیم دیگه نیازی به شرط 4 نیست:105:

tst 2012 Puerto Rico
استقرا. فرض کنید f روی اعداد 1 تا 2[SUP]n[/SUP] همانی باشه. ثابت می کنیم روی اعدا 1 تا 2[SUP]n+1 [/SUP]هم همانیه. واضحه که f(2[SUP]n[/SUP])=2[SUP]n[/SUP]
حالا فرض کنید k بین 2[SUP]n[/SUP] و 2[SUP]n+1[/SUP] باشه. میخوایم ثابت کنیم fاش خودشه. اگه زوج بود که چون f نصفش بین یک تا 2[SUP]n[/SUP] هست پس f نصفش خود نصفشه. پس f(k)=k و اگه غرد بود یکی بیشتر و یکی کمترش زوجند پس fاشون خودشونه پس طبق فرض 4 داریم f(k)=k
 

math

New Member
ارسال ها
1,129
لایک ها
1,096
امتیاز
0
پاسخ : ماراتن جبر (سطح پیشرفته)

این راه منه درسته؟؟؟؟

میدانیم جواب های معادله زیر به شکل
است



پس داریم
پس تابع همانی است
 

alich100

New Member
ارسال ها
202
لایک ها
90
امتیاز
0
پاسخ : ماراتن جبر (سطح پیشرفته)

چرا جوابا به اون شکله؟
پیوسته نیست یعنی از توابع کوشی نمی تونی استفاده کنی
البته فک کنم!
 

ash1374

New Member
ارسال ها
253
لایک ها
422
امتیاز
0
پاسخ : ماراتن جبر (سطح پیشرفته)

این راه منه درسته؟؟؟؟

میدانیم جواب های معادله زیر به شکل
است

نه لزوما به اون شکل نست. مثلا اگه p[SUB]n[/SUB] رو nامین عدد اول بگیریم تعریف کنید

حالا f هر عدد به صورت یکتا تعیین میشه و این f ضربی هم هست ولی به اون فرم توانی نیست.
 

Aref

New Member
ارسال ها
1,262
لایک ها
1,008
امتیاز
0
پاسخ : ماراتن جبر (سطح پیشرفته)

سلام
می خواستم بگم چند تا از نامساوی مثل این بزارید بد نیست

متغیر ها مثبتن
به توان سه برسونید، بعد جملات خوب رو بیارید اینور به توان دو برسونید، بعد am-gm بزنید
 

Aref

New Member
ارسال ها
1,262
لایک ها
1,008
امتیاز
0
پاسخ : ماراتن جبر (سطح پیشرفته)

راستی اینجا نامساوی نزارید. برای نامساوی دو سه تا ماراتن هست که شوت شده پایین. توی انجمن ها بگردید پیدا می کنید.
 

amircaracas

New Member
ارسال ها
36
لایک ها
16
امتیاز
0
پاسخ : ماراتن جبر (سطح پیشرفته)

البته یه لم هست که میگه اگر یه تابع صعودی باشه و ضربی هم باشه میشه از کوشی ضربی استفاده کرد.
 

Aref

New Member
ارسال ها
1,262
لایک ها
1,008
امتیاز
0
پاسخ : ماراتن جبر (سطح پیشرفته)

البته یه لم هست که میگه اگر یه تابع صعودی باشه و ضربی هم باشه میشه از کوشی ضربی استفاده کرد.
اون لم برای توابع حقیقی درسته.
 

math

New Member
ارسال ها
1,129
لایک ها
1,096
امتیاز
0
پاسخ : ماراتن جبر (سطح پیشرفته)

خوب اینم یک حالت خاص حقیقیه دیگه!!!!

میشه اثبات لم رو هم بذارید :53:
 

Aref

New Member
ارسال ها
1,262
لایک ها
1,008
امتیاز
0
پاسخ : ماراتن جبر (سطح پیشرفته)

:21:
-----------------
این چیزی که گفتید فقط وقتی درسته که یه توسیعی از اون تابع صحیح به اعداد حقیقی وجود داشته باشه که اون خاصیت های گفته شده رو داشته باشه. اعداد حقیقی خواصی که دارند با اعداد صحیح فرق می کنه.
 
بالا